Could we simplify the log determinant's concavity proof?











up vote
1
down vote

favorite












The function $f(X) Rightarrow log det X$ is concave as shown here.
However, I was wondering if we could simplify the proof suggested.



When we compute : $g(t) = logdet(Z + tV)$, why not just saying that:



$$begin{aligned}
g(t) &=
logdet(Z)(I+tZ^{-1}V) \
&= sum_i log(1+tlambda_i). + logdet Z
end{aligned}$$

with $(lambda_i)$ the eigenvalues of $Z^{-1}V$?










share|cite|improve this question






















  • Are your matrices positive semidefinite or something? And yes, I agree that the introduction of $Z^{1/2}$ is unneeded.
    – darij grinberg
    Oct 10 at 16:37












  • OK thanks. I think that the problem is not defined if the matrices are not positive definite.
    – pl-94
    Oct 10 at 16:48















up vote
1
down vote

favorite












The function $f(X) Rightarrow log det X$ is concave as shown here.
However, I was wondering if we could simplify the proof suggested.



When we compute : $g(t) = logdet(Z + tV)$, why not just saying that:



$$begin{aligned}
g(t) &=
logdet(Z)(I+tZ^{-1}V) \
&= sum_i log(1+tlambda_i). + logdet Z
end{aligned}$$

with $(lambda_i)$ the eigenvalues of $Z^{-1}V$?










share|cite|improve this question






















  • Are your matrices positive semidefinite or something? And yes, I agree that the introduction of $Z^{1/2}$ is unneeded.
    – darij grinberg
    Oct 10 at 16:37












  • OK thanks. I think that the problem is not defined if the matrices are not positive definite.
    – pl-94
    Oct 10 at 16:48













up vote
1
down vote

favorite









up vote
1
down vote

favorite











The function $f(X) Rightarrow log det X$ is concave as shown here.
However, I was wondering if we could simplify the proof suggested.



When we compute : $g(t) = logdet(Z + tV)$, why not just saying that:



$$begin{aligned}
g(t) &=
logdet(Z)(I+tZ^{-1}V) \
&= sum_i log(1+tlambda_i). + logdet Z
end{aligned}$$

with $(lambda_i)$ the eigenvalues of $Z^{-1}V$?










share|cite|improve this question













The function $f(X) Rightarrow log det X$ is concave as shown here.
However, I was wondering if we could simplify the proof suggested.



When we compute : $g(t) = logdet(Z + tV)$, why not just saying that:



$$begin{aligned}
g(t) &=
logdet(Z)(I+tZ^{-1}V) \
&= sum_i log(1+tlambda_i). + logdet Z
end{aligned}$$

with $(lambda_i)$ the eigenvalues of $Z^{-1}V$?







matrices complex-analysis determinant






share|cite|improve this question













share|cite|improve this question











share|cite|improve this question




share|cite|improve this question










asked Oct 10 at 16:34









pl-94

1184




1184












  • Are your matrices positive semidefinite or something? And yes, I agree that the introduction of $Z^{1/2}$ is unneeded.
    – darij grinberg
    Oct 10 at 16:37












  • OK thanks. I think that the problem is not defined if the matrices are not positive definite.
    – pl-94
    Oct 10 at 16:48


















  • Are your matrices positive semidefinite or something? And yes, I agree that the introduction of $Z^{1/2}$ is unneeded.
    – darij grinberg
    Oct 10 at 16:37












  • OK thanks. I think that the problem is not defined if the matrices are not positive definite.
    – pl-94
    Oct 10 at 16:48
















Are your matrices positive semidefinite or something? And yes, I agree that the introduction of $Z^{1/2}$ is unneeded.
– darij grinberg
Oct 10 at 16:37






Are your matrices positive semidefinite or something? And yes, I agree that the introduction of $Z^{1/2}$ is unneeded.
– darij grinberg
Oct 10 at 16:37














OK thanks. I think that the problem is not defined if the matrices are not positive definite.
– pl-94
Oct 10 at 16:48




OK thanks. I think that the problem is not defined if the matrices are not positive definite.
– pl-94
Oct 10 at 16:48










1 Answer
1






active

oldest

votes

















up vote
0
down vote



accepted










Actually, we can not simplify the proof. Indeed, $Z^{-1}V$ is not necessarily symmetric, and thus, the decomposition to the eigenvalues is not guaranteed.



This explains the use of the trick with $Z^{1/2}$






share|cite|improve this answer





















    Your Answer





    StackExchange.ifUsing("editor", function () {
    return StackExchange.using("mathjaxEditing", function () {
    StackExchange.MarkdownEditor.creationCallbacks.add(function (editor, postfix) {
    StackExchange.mathjaxEditing.prepareWmdForMathJax(editor, postfix, [["$", "$"], ["\\(","\\)"]]);
    });
    });
    }, "mathjax-editing");

    StackExchange.ready(function() {
    var channelOptions = {
    tags: "".split(" "),
    id: "69"
    };
    initTagRenderer("".split(" "), "".split(" "), channelOptions);

    StackExchange.using("externalEditor", function() {
    // Have to fire editor after snippets, if snippets enabled
    if (StackExchange.settings.snippets.snippetsEnabled) {
    StackExchange.using("snippets", function() {
    createEditor();
    });
    }
    else {
    createEditor();
    }
    });

    function createEditor() {
    StackExchange.prepareEditor({
    heartbeatType: 'answer',
    convertImagesToLinks: true,
    noModals: true,
    showLowRepImageUploadWarning: true,
    reputationToPostImages: 10,
    bindNavPrevention: true,
    postfix: "",
    imageUploader: {
    brandingHtml: "Powered by u003ca class="icon-imgur-white" href="https://imgur.com/"u003eu003c/au003e",
    contentPolicyHtml: "User contributions licensed under u003ca href="https://creativecommons.org/licenses/by-sa/3.0/"u003ecc by-sa 3.0 with attribution requiredu003c/au003e u003ca href="https://stackoverflow.com/legal/content-policy"u003e(content policy)u003c/au003e",
    allowUrls: true
    },
    noCode: true, onDemand: true,
    discardSelector: ".discard-answer"
    ,immediatelyShowMarkdownHelp:true
    });


    }
    });














     

    draft saved


    draft discarded


















    StackExchange.ready(
    function () {
    StackExchange.openid.initPostLogin('.new-post-login', 'https%3a%2f%2fmath.stackexchange.com%2fquestions%2f2950262%2fcould-we-simplify-the-log-determinants-concavity-proof%23new-answer', 'question_page');
    }
    );

    Post as a guest















    Required, but never shown

























    1 Answer
    1






    active

    oldest

    votes








    1 Answer
    1






    active

    oldest

    votes









    active

    oldest

    votes






    active

    oldest

    votes








    up vote
    0
    down vote



    accepted










    Actually, we can not simplify the proof. Indeed, $Z^{-1}V$ is not necessarily symmetric, and thus, the decomposition to the eigenvalues is not guaranteed.



    This explains the use of the trick with $Z^{1/2}$






    share|cite|improve this answer

























      up vote
      0
      down vote



      accepted










      Actually, we can not simplify the proof. Indeed, $Z^{-1}V$ is not necessarily symmetric, and thus, the decomposition to the eigenvalues is not guaranteed.



      This explains the use of the trick with $Z^{1/2}$






      share|cite|improve this answer























        up vote
        0
        down vote



        accepted







        up vote
        0
        down vote



        accepted






        Actually, we can not simplify the proof. Indeed, $Z^{-1}V$ is not necessarily symmetric, and thus, the decomposition to the eigenvalues is not guaranteed.



        This explains the use of the trick with $Z^{1/2}$






        share|cite|improve this answer












        Actually, we can not simplify the proof. Indeed, $Z^{-1}V$ is not necessarily symmetric, and thus, the decomposition to the eigenvalues is not guaranteed.



        This explains the use of the trick with $Z^{1/2}$







        share|cite|improve this answer












        share|cite|improve this answer



        share|cite|improve this answer










        answered 14 hours ago









        pl-94

        1184




        1184






























             

            draft saved


            draft discarded



















































             


            draft saved


            draft discarded














            StackExchange.ready(
            function () {
            StackExchange.openid.initPostLogin('.new-post-login', 'https%3a%2f%2fmath.stackexchange.com%2fquestions%2f2950262%2fcould-we-simplify-the-log-determinants-concavity-proof%23new-answer', 'question_page');
            }
            );

            Post as a guest















            Required, but never shown





















































            Required, but never shown














            Required, but never shown












            Required, but never shown







            Required, but never shown

































            Required, but never shown














            Required, but never shown












            Required, but never shown







            Required, but never shown







            Popular posts from this blog

            Can a sorcerer learn a 5th-level spell early by creating spell slots using the Font of Magic feature?

            Does disintegrating a polymorphed enemy still kill it after the 2018 errata?

            A Topological Invariant for $pi_3(U(n))$